3
$\begingroup$

Is it possible to prove that a $Beta(\alpha,\beta)$ distributed random variable is always stochastically dominated by a $Beta(\alpha',\beta')$ if $\alpha'>\alpha$ and $\beta'<\beta$? I've read it in an article but I couldn't find any bibliography about. The proof is straightforward whenever $\alpha'+\beta'=\alpha+\beta$ using the Beta-Binomial identity and exploiting the fact that for $n ≥ m, q ≥ p ⇒ Bin(n, q) \succeq Bin(m, p)$. But I couldn't prove this fact whenever such a bound isn't enforced.

$\endgroup$

1 Answer 1

3
$\begingroup$

$\newcommand\al\alpha\newcommand\be\beta$Let $f_{\al,\be}$ be the pdf of the beta distribution with parameters $\al,\be$. Here we have a monotone likelihood ratio, in the sense that $$\frac{f_{\al',\be'}(x)}{f_{\al,\be}(x)} =c_{\al,\be,\al',\be'}\, x^{\al'-\al}(1-x)^{\be'-\be}$$ is increasing in $x\in(0,1)$ if $\al'>\al$ and $\be'<\be$, where $c_{\al,\be,\al',\be'}$ is a positive real number not depending on $x$. So, the desired stochastic domination follows.

$\endgroup$

You must log in to answer this question.

Start asking to get answers

Find the answer to your question by asking.

Ask question

Explore related questions

See similar questions with these tags.